Second order ODE into a system of first order ODEs

Click For Summary
The discussion revolves around converting the second-order ordinary differential equation (ODE) of a harmonic oscillator into a system of first-order ODEs. The user correctly identifies the variables X1 and X2 to represent position and velocity, respectively, but expresses concern about the forcing function f(t) and its impact on the solution. A participant points out that the forcing term seems to have been omitted in the user's equations, suggesting it should not be disregarded. The user acknowledges this oversight and considers the need for initial conditions in their solution. The conversation emphasizes the importance of including all terms in the equations when transitioning between ODE forms.
whatisgoingon
Messages
9
Reaction score
0

Homework Statement


The harmonic oscillator's equation of motion is:

x'' + 2βx' + ω02x = f

with the forcing of the form f(t) = f0sin(ωt)

The Attempt at a Solution



So I got:
X1 = x
X1' = x' = X2
X2 = x'
X2' = x''
∴ X2' = -2βX2 - ω02X1 + sin(ωt)

The function f(t) is making me doubt this answer because I have to take into account f0 and it just disappears in the solution.

(For context I have to put it into a system of first order ODEs because I have to code it into python and plot the results with the given parameters.)

Am I on the right track? Or am I missing anything?
 
Physics news on Phys.org
whatisgoingon said:
it just disappears in the solution
Looks to me you made it disappear in the equations already.
 
BvU said:
Looks to me you made it disappear in the equations already.
Oh so I shouldn't have taken it out in the first place then? That makes sense thanks! Other than that, am I missing anything else?
 
Initial conditions ?
 
Question: A clock's minute hand has length 4 and its hour hand has length 3. What is the distance between the tips at the moment when it is increasing most rapidly?(Putnam Exam Question) Answer: Making assumption that both the hands moves at constant angular velocities, the answer is ## \sqrt{7} .## But don't you think this assumption is somewhat doubtful and wrong?

Similar threads

Replies
4
Views
1K
  • · Replies 2 ·
Replies
2
Views
2K
  • · Replies 2 ·
Replies
2
Views
2K
  • · Replies 5 ·
Replies
5
Views
2K
  • · Replies 4 ·
Replies
4
Views
8K
Replies
2
Views
2K
  • · Replies 14 ·
Replies
14
Views
3K
  • · Replies 4 ·
Replies
4
Views
2K
  • · Replies 6 ·
Replies
6
Views
2K
Replies
3
Views
2K